Hardware Upgrade Forum

Hardware Upgrade Forum (https://www.hwupgrade.it/forum/index.php)
-   Scienza e tecnica (https://www.hwupgrade.it/forum/forumdisplay.php?f=91)
-   -   [Official Thread]Richieste d'aiuto in MATEMATICA: postate qui! (https://www.hwupgrade.it/forum/showthread.php?t=1221191)


superchicco95 09-03-2015 06:39

Scusate, sono sempre io con le mie rette. Qualcuno usa sullo smartphone l'app QuickGraph? Anche lì non so come scrivere la retta in 3D, ho cercato su google, ma non trovo niente

ciriccio 09-03-2015 09:01

puoi scrivere i 2 piani e ruotando col dito poi visualizzare la loro intersezione...
per "eliminare" i due piani magari trovi qualcosa pagando nelle opzioni avanzate boh...

thewebsurfer 16-03-2015 13:14

salve ragazzi, qualcuno mi sa dare una spiegazione al fatto che in ambito "Probabilità e statistica" la varianza è del tipo
σ^2=1/n * sommatoria da 1 a n di (x_i - E(X))^2

(E(X) valore medio, x_i insieme di n dati)

quindi si divide per n, mentre in altri ambiti (analisi dei dati) si divide per 1/(n-1)?

Ziosilvio 16-03-2015 15:23

Quote:

Originariamente inviato da thewebsurfer (Messaggio 42262685)
salve ragazzi, qualcuno mi sa dare una spiegazione al fatto che in ambito "Probabilità e statistica" la varianza è del tipo
σ^2=1/n * sommatoria da 1 a n di (x_i - E(X))^2

(E(X) valore medio, x_i insieme di n dati)

quindi si divide per n, mentre in altri ambiti (analisi dei dati) si divide per 1/(n-1)?

La prima quantità è la varianza "vera e propria", usata in calcolo delle probabilità, in cui E(X) è il valore atteso (media teorica).
La seconda, si chiama "varianza campionaria corretta", e viene adoperata in statistica, con E(X) pari non al valore atteso (che, in generale, non è noto) ma alla media campionaria, ossia alla media aritmetica dei valori raccolti.
Il motivo per questo nome, è che il valore atteso della varianza campionaria corretta è proprio la varianza: cosicchè la prima è uno stimatore non distorto della seconda.

thewebsurfer 16-03-2015 17:45

grazie mille, con le poche info che ti ho dato hai centrato il discorso.
Ho studiato 'probabilità e statistica' ed ora 'analisi dei dati' in modo consequenziale, ed è quasi un trauma vedere che concetti simili abbiano applicazioni che sembrano simili ma in realtà sono molto diverse..
mi sembra di affrontare una cosa tipo "ok fin'ora hai saputo che l'addizione si fa così, però abbiamo scherzato in realtà si fa in quest'altro modo..":D

Nel merito, ok, E(X) ha due significati diversi nelle due varianze, ma ancora non mi è chiarissimo l'n-1, al di là di una non meglio precisata "distorsione"

thewebsurfer 24-03-2015 11:39

1 Allegato(i)
altra domanda, sempre in ambito di statistica descrittiva, il tema è la Concentrazione
non riesco a capire cosa siano p_i e q_i. Anche perché, ignoranto la formula analitica e leggendo solo i paragrafi hanno stessa definizione "percentuale cumulata dei primi i possessori del carattere":confused:

superchicco95 04-04-2015 17:10

Sto studiando Analisi 2 e mi è stato dato un esercizio in cui devo dimostrare che la funzione definita
f(x,y)=xy(x^2-y^2)/(x^2+y^2) per (x,y)!=(0,0) ed f(x,y)=0 per (x,y)=0
ammette derivate parziali seconde in (0,0), ma non vale il teorema di Schwarz, quindi le miste sono diverse.
Ho provato la differenziabilità in (0,0) e ho calcolato le derivate parziali prime. Poi volevo calcolare le miste usando la definizione di derivata direzionale, ma mi risultano entrambe pari a zero.
Ho poi consultato la pagina di wikipedia sul teorema di Schwarz e viene proprio riportato l'esercizio che devo svolgere io.
http://it.wikipedia.org/wiki/Teorema_di_Schwarz
Lì le derivate parziali miste (calcolate tramite derivata direzionale) risultano pari a -1 e 1, ma non so come le abbia fatte saltare fuori! E sì che non e ho fatte poche di derivate direzionali... anzi...
Qualche buon'anima mi può spiegare passo per passo come fare e dirmi magari dove mi perdo? Soprattutto in riferimento a quella pagina di wikipedia che riporta proprio il mio esempio.
Grazie!

luigimitico 04-04-2015 17:47

hai difficolta' a fare la derivata parziale? In quale : in quella rispetto a x o rispetto a y?
Quella è una funzione composta da due parti la prima è xy mentre la seconda è (x^2-y^2)/(x^2+y^2) . Ovvio che devi fare una volta la derivata della prima per la seconda non derivata e poi la derivata della seconda per la prima non derivata. questo lo devi derivare una volta rispetto a x e una volta rispetto a y.
prova a rifarti le derivate parziali, non sono esageratamente difficili, attento a ricordarti che nella seconda funzione (x^2-y^2)/(x^2+y^2) devi fare il quadrato del denominatore.

se vuoi sapere come il limite per k->0 fa zero devi fare la cosi:
lim k->0 di f_x(0,0)= (k(0-k^2)/k^2)/k=(-k^3/k^2)/k=-1

superchicco95 04-04-2015 17:59

Non ho problemi a calcolare il gradiente, infatti le derivate parziali prime mi vengono come su quella di pagina di wikipedia. Il problema è quando vado a fare le derivate parziali seconde miste tramite definizione di derivata direzionale. A me risultano entrambe pari a zero (quindi uguali e non diverse come dice di dimostrare la consegna dell'esercizio), mentre su quella pagina risultano pari a -1 e 1. Quindi evidentemente mi perdo qualcosa nella derivata direzionale, anche se mi sembra molto molto strano

luigimitico 04-04-2015 22:02

prova a scrivere il calcolo che fai e vediamo deve è l'errore.

superchicco95 05-04-2015 10:40

Scusate, non ho parole... Dimenticavo di dividere per k nel limite... Lo sapevo che non mi sarei dovuto mettere a studiare col mal di testa. Grazie comunque!

xenom 08-04-2015 20:03

devo dare analisi 1... io ho dei grossi problemi con i limiti, ovviamente sulle forme indeterminate.
Il problema è che ci sono 7-8 metodi per risolverli e non riesco mai a capire quale funziona meglio. in particolare non capisco se in realtà tutti i metodi sono validi e portano allo stesso risultato..
Non capisco cosa sbaglio ma pur facendo passaggi algebrici di semplificazione che mi sembrano corretti, non arrivo al risultato corretto.

Cioè con i limiti è facile sbagliare perché ogni risultato è potenzialmente corretto.
mi date qualche dritta su come affrontare un limite "complesso"?

PS: sto affrontando analisi 1 quasi da autodidatta quindi sto facendo un po' di fatica :help: :stordita:

luigimitico 08-04-2015 20:29

ciao spero che tu abbia chiaro il concetto di limite.
Purtroppo le varie forme di risoluzione sono utili quando servono. Alcuni esercizi si risolvono facilmente con un metodo e con un altro non riesci proprio. non c'e' una regola buona per tutto. Fai tanti esercizi, parti da quelli piu' semplici e poi vai a quelli piu' difficili.
Cerca dei libri che presentato tanti esercizi e risolvili. Una cosa molto utile è quella di fare esercizi con altri se possibile cosi ti confronti e capisci quali sono i tuoi punti deboli e magari qualcuno riesce a darti un piccolo aiuto per poter andare avanti.

Lampo89 08-04-2015 20:34

Quote:

Originariamente inviato da xenom (Messaggio 42348548)
devo dare analisi 1... io ho dei grossi problemi con i limiti, ovviamente sulle forme indeterminate.
Il problema è che ci sono 7-8 metodi per risolverli e non riesco mai a capire quale funziona meglio. in particolare non capisco se in realtà tutti i metodi sono validi e portano allo stesso risultato..
Non capisco cosa sbaglio ma pur facendo passaggi algebrici di semplificazione che mi sembrano corretti, non arrivo al risultato corretto.

Cioè con i limiti è facile sbagliare perché ogni risultato è potenzialmente corretto.
mi date qualche dritta su come affrontare un limite "complesso"?

PS: sto affrontando analisi 1 quasi da autodidatta quindi sto facendo un po' di fatica :help: :stordita:

Immagino che non siano un problema forme di indecisioni risolvibili con confronti tra infiniti, per esperienza credo ti riferisca alle applicazioni dei
limiti notevoli .. Il mio consiglio è di impararti bene gli sviluppi di taylor e mac laurin più importarti, e svolgere tutto usando opiccoli e robe simili..
e fai molti esercizi

prova a ragionare su questo:

lim x->1 (sen(pi*x^3) + 3*pi*(x-1)^2)/(x - x^(1/5))

superchicco95 09-04-2015 10:00

[Official Thread]Richieste d'aiuto in MATEMATICA: postate qui!
 
Qualcuno mi può dimostrare la non esistenza di questo limite (almeno così dice WolframAlpha)? Io non riesco a beccare la giusta direzione.
lim (5x+3y)/(x^2+y^2) as (x,y)->(0,0)
Grazie!

Ziosilvio 09-04-2015 12:35

Quote:

Originariamente inviato da superchicco95 (Messaggio 42349979)
Qualcuno mi può dimostrare la non esistenza di questo limite (almeno così dice WolframAlpha)? Io non riesco a beccare la giusta direzione.
lim (5x+3y)/(x^2+y^2) as (x,y)->(0,0)
Grazie!

Se il limite esiste, allora deve esistere, ed essere lo stesso, anche lungo traiettorie particolari.
Che cosa succede per x > 0 ed y = 0? Che cosa succede per x < 0 ed y = 0?

superchicco95 09-04-2015 12:40

Infatti, ma non riesco a dimostrare la non esistenza tramite direzioni diverse. Qualcuno me lo può fare?

Lampo89 10-04-2015 20:43

Quote:

Originariamente inviato da superchicco95 (Messaggio 42350765)
Infatti, ma non riesco a dimostrare la non esistenza tramite direzioni diverse. Qualcuno me lo può fare?

Zio Silvio ti ha suggerito una direzione particolare da controllare ... in ogni caso, il comportamento è qualitativamente lo stesso avvicinandosi a (0,0) lungo qualsiasi rette ...

peppozzolo 15-05-2015 15:52

ragazzi mi sapreste aiutare con il dominio della funzione

(3*arcocotan(x) -pi)^pi

il risultato è ]-inf, -(3^1/2)/3] u [0,+inf[

mi ritrovo la prima parte ponendo tutta la parte in parentesi >=0 e facendo e passando alla cotangente, ma non riesco a capire da dove esce la seconda parte ,perche [0,+inf[ ?
grazie a tutti per l'attenzione

Lampo89 15-05-2015 21:05

Quote:

Originariamente inviato da peppozzolo (Messaggio 42479180)
ragazzi mi sapreste aiutare con il dominio della funzione

(3*arcocotan(x) -pi)^pi

il risultato è ]-inf, -(3^1/2)/3] u [0,+inf[

mi ritrovo la prima parte ponendo tutta la parte in parentesi >=0 e facendo e passando alla cotangente, ma non riesco a capire da dove esce la seconda parte ,perche [0,+inf[ ?
grazie a tutti per l'attenzione

io non capisco perché quella roba lì non dovrebbe essere definita tra - sqrt(3)/3 e 0...
perché imponi che la base debba essere positiva? (probabile che sia una cosa che non ricordo )

peppozzolo 15-05-2015 22:00

in pratica avendo esponente reale si pone:
3*arcocotan(x) -pi>=0
che per la decrescenza della cotangente si cambia segno di disequazione(giusto?) e si ha:
x<=cotangente(pi/3) da cui x<= - sqrt(3)/3 da cui la prima parte del dominio, la seconda non me la so spiegare

Lampo89 16-05-2015 11:03

Quote:

Originariamente inviato da peppozzolo (Messaggio 42480419)
in pratica avendo esponente reale si pone:
3*arcocotan(x) -pi>=0
che per la decrescenza della cotangente si cambia segno di disequazione(giusto?) e si ha:
x<=cotangente(pi/3) da cui x<= - sqrt(3)/3 da cui la prima parte del dominio, la seconda non me la so spiegare

se prendi una x tra 0 e inf abbastanza grande dovresti vedere che la base della potenza diventa negativa... credo che il risultato riportato sia sbagliato e che il tuo sia corretto ....
by the way, se consideri la stessa funzione, invece che a valori reali, a valori complessi credo che il dominio dovrebbe coincidere con R

peppozzolo 16-05-2015 11:19

risolvendo con wolfram da ragione al libro non so proprio che pesci prendere. Troppo presto per analisi complesse :D

Lampo89 16-05-2015 12:02

Quote:

Originariamente inviato da peppozzolo (Messaggio 42481399)
risolvendo con wolfram da ragione al libro non so proprio che pesci prendere. Troppo presto per analisi complesse :D

la arco cotangente di un numero molto grande (diciamo 10000) è un numero molto piccolo, << 1 . se ad esso sottrai pigreco, ottieni un numero negativo
Da quello dici, non puoi elevare un numero negativo a potenza irrazionale, per cui il campo di esistenza che riporta il tuo libro è sbagliato..
edit:
forse è meglio che mi ristudio, nei ritagli di tempo, le funzioni trigonometriche:
è vero, l'arcocotangente è una funzione monotona decrescente ma NON è continua in zero !! lo vedi facile se fai un grafico con wolfram alpha: grafica il membro di sx della disequazione che devi risolvere, e poi trova le che x la cui ordinata sta sopra la retta y = 0 (il solito metodo per risolvere graficamente disequazioni) ... insomma ti becchi soluzione 0 < x < 1/sqrt(3) (a meno di uguali) , che non è né quella che riporta il libro, né quello che hai trovato, ma è anche la soluzione che wolfram riporta ...
perché cot(pi/3) = -1/sqrt(3) ??? cot(pi/3) = 1/tan(pi/3) = 1/sqrt(3)

peppozzolo 16-05-2015 12:48

scusa ma la funzione non è questa?


mi sembra continua in zero, forse hai visto su wolfram dove secondo me prendono una restrizione del dominio diversa, forse da quello nasce il disguido?

Lampo89 16-05-2015 14:34

Quote:

Originariamente inviato da peppozzolo (Messaggio 42481715)
scusa ma la funzione non è questa?


mi sembra continua in zero, forse hai visto su wolfram dove secondo me prendono una restrizione del dominio diversa, forse da quello nasce il disguido?

Ribadisco... Un motivo in più per me di ripassare queste cose.. In effetti wiki definisce la funzione inversa della cot tra 0 e pi ( e non tra - pi/2 e pi/2 come fa wolfram, o su mathworld) e questo spiega la differenza.. Ma comunque, la soluzione del tuo libro non mi torna ..

luigimitico 16-05-2015 19:32

ma sei sicuro che la elevazione a potenza sia pi-greco?

peppozzolo 17-05-2015 10:11

Sisi l'esponente é pi greco. Il mistero si infittisce :-)

luigimitico 17-05-2015 16:00

La seconda parte del dominio
[0,+inf[
non è inclusa totalmente, basta semplicemente sostituire un qualunque numero di questo intervallo maggiore di 1/sqrt(3) (cioe circa 0,6) nella funzione per ottenere numeri negativi. ad esempio per x=1 otterresti
3*pi/4 - pi = -(1/4)pi che è negativo quindi non adatto ad essere elevato a potenza reale
per valori ancora piu' grandi il numero negativo cresce in valore assoluto.
Quindi i numeri accettabili sono quelli da
(-inf,1/sqrt(3)) oppure scritto come (-inf, sqrt(3)/3 )

peppozzolo 17-05-2015 17:58

mi sa che é l'unica spiegazione, grazie a tutti :-)

Ziosilvio 17-05-2015 18:06

Quote:

Originariamente inviato da peppozzolo (Messaggio 42479180)
ragazzi mi sapreste aiutare con il dominio della funzione

(3*arcocotan(x) -pi)^pi

il risultato è ]-inf, -(3^1/2)/3] u [0,+inf[

mi ritrovo la prima parte ponendo tutta la parte in parentesi >=0 e facendo e passando alla cotangente, ma non riesco a capire da dove esce la seconda parte ,perche [0,+inf[ ?
grazie a tutti per l'attenzione

Ho il sospetto che il libro consideri il dominio della cotangente non come (0, pi), ma come (-pi/2,0) union (0, pi/2]: e che lo stesso faccia Wolfram Alpha.
Lo dico perché, ad un'occhiata veloce, GNU Octave sembra pensarla esattamente in questo modo :eek:

Ah: e poi, ovviamente, c'è la possibilità che questi software considerino qualche prolungamento analitico dell'arcocotangente, sviluppato in serie di potenze...

superchicco95 24-05-2015 16:37

Nel venire qui per postare un nuovo dubbio, mi sono accorto di non aver neanche ringraziato per il suggerimento in quel limite che avevo postato ormai due mesi fa. Evidentemente tapatalk non aveva inviato il messaggio. Quindi grazie!
Adesso sto studiando le forme differenziali, campi e potenziali. Ho però un grosso dubbio. Io so che una forma differenziale è esatta se e solo se il campo è conservativo, di conseguenza esiste una funzione potenziale; e so anche che se una forma differenziale è esatta allora è chiusa. L'implicazione contraria vale solo per i domini semplicemente connessi, per i quali posso verificare se la forma differenziale è esatta, provandone la chiusura con le derivate incrociate. E fin qui ci sono.
Il problema si pone quando voglio verificare se una forma differenziale è esatta in un dominio che non è semplicemente connesso. Per esempio, prendiamo R^2 privato dell'origine: è giusto calcolare l'integrale curvilineo di seconda specie sulla circonferenza chiusa centrata nell'origine, di raggio 1? Se vedo che non è nulla (come dovrebbe essere dato che è un percorso chiuso), allora è corretto concludere che la forma non è esatta?
Se considerassi invece come dominio R^2 con y!=0, come mi dovrei comportare?
Grazie già anticipatamente!

Ziosilvio 24-05-2015 21:32

Se ho capito bene, "R^2 con y!=0" significa "il piano, privato dell'asse delle ascisse".
Questo dominio non solo non è semplicemente connesso: non è affatto connesso! Però, ciascuna delle sue componenti connesse (sottoinsiemi connessi massimali) è semplicemente connessa. Quindi, una forma differenziale che sia chiusa in una di queste componenti connesse, sarà anche esatta in quella componente connessa.

superchicco95 25-05-2015 07:29

Perfetto, avevo anch'io fatto un ragionamento del genere, ma non ero sicuro che fosse giusto. Grazie!
Invece quando è che devo calcolarmi la funzione potenziale costruendo una curva (tipicamente ad L)? Mi riferisco al classico esempio della forma differenziale -y/(x^2+y^2)dx+x/(x^2+y^2)dy. Se come dominio prendo R^2 escluso l'origine vedo che non è esatta calcolando l'integrale curvilineo di seconda specie sulla circonferenza centrata nell'origine. Se considero R^2, x>0 a questo punto so che è esatta e mi calcolo la funzione potenziale usando l'integrale curvilineo di seconda specie su una curva a L. Non ho ben capito perché devo usare questo procedimento, e quindi quando devo usarlo...

Ziosilvio 25-05-2015 09:21

Il punto è che le forme differenziali esatte in un dominio sono precisamente le forme chiuse il cui integrale lungo un qualsiasi cammino chiuso contenuto nel dominio è nullo.
Se il dominio è semplicemente connesso, allora sei sicuro che l'integrale di una forma chiusa lungo un cammino chiuso è nullo: questo perché l'integrale di una forma differenziale chiusa lungo un cammino chiuso è lo stesso che su qualsiasi altro cammino chiuso omotopicamente equivalente al primo, e in un dominio semplicemente connesso ogni cammino chiuso è omotopicamente equivalente ad un punto.
Se il dominio non è semplicemente connesso, questo trucco non funziona più.

superchicco95 25-05-2015 09:38

E quindi devo calcolarmi la funzione potenziale come integrale curvilineo di seconda specie ogni volta che non ho un dominio semplicemente connesso? E dovrei poi controllare se il suo gradiente corrisponde al campo per capire se è conservativo (e quindi una forma esatta), giusto?

superchicco95 03-06-2015 21:14

Ho un'altra domanda, forse stupida. Mi viene data una forma differenziale su un dominio semplicemente connesso di cui ho già verificato l'esattezza e ho trovato la funzione potenziale. Se mi viene chiesto di calcolare l'integrale curvilineo di seconda specie lungo la curva ottenuta dall'intersezione di z=x^2+4y^2 e z=3x-2y da (0,0,0) a (1,1/2,2), posso sfruttare la funzione potenziale sostituendo gli estremi giusto? Ma se la forma non fosse stata esatta, e avessi dovuto calcolarlo da definizione, come avrei parametrizzato quella curva? Grazie ancora!

thewebsurfer 11-10-2015 18:01

salve, qualcuno mi sa dire cos'è la risposta impulsiva di un segnale (in ambito "analisi dei dati e segnali")?

xenom 11-10-2015 18:35

sono in difficoltà con la convergenza uniforme delle serie di funzioni... non ho ancora ben capito come affrontare gli esercizi.
Se mi ritrovo una serie di funzioni qualsiasi, dimostro l'eventuale convergenza assoluta nell'eventuale dominio di convergenza e fin qua ok, ma poi quando devo trovare se converge uniformemente non ho la minima idea da dove cominciare. E non ho capito la storia del maggiorante del |fn(x) - f(x)| :stordita:

Ziosilvio 11-10-2015 19:04

Per dimostrare che una serie di funzioni f_n converge uniformemente ad una somma S in un certo insieme U, devi far vedere che, detta S_n la somma parziale dei termini fino all'n-esimo, comunque preso e > 0, puoi trovare k >= 0 dipendente da e ma non da x, tale che, se n > k, allora |S_n(x) - S(x)| < e per ogni x in U.

Tipico esempio con le serie di potenze:
La serie di funzioni f_n(x) = x^n converge uniformemente nell'intervallo (-1/2, 1/2) alla funzione S(x) = 1 / (1-x).
Questo perché la serie resto è:



e puoi maggiorare in modulo x^{n+1} con 1/2^{n+1} ed S(x) con 2.


Tutti gli orari sono GMT +1. Ora sono le: 04:29.

Powered by vBulletin® Version 3.6.4
Copyright ©2000 - 2024, Jelsoft Enterprises Ltd.
Hardware Upgrade S.r.l.